Download as pdf or txt
Download as pdf or txt
You are on page 1of 13

4/24/2021 Prelim Exam: Attempt review

Home / My courses / UGRD-MATH6330-2033T / Week 4: Preliminary Examination / Prelim Exam

Started on Saturday, 24 April 2021, 4:49 PM


State Finished
Completed on Saturday, 24 April 2021, 5:06 PM
Time taken 16 mins 19 secs
Grade 19.00 out of 30.00 (63%)

Question 1
Incorrect

Mark 0.00 out of 1.00

For practical reasons, the absolute error is usually more meaningful than the relative error.

Select one:
True 

False

Question 2
Correct

Mark 1.00 out of 1.00

The relative error is related to the approximate value rather than to the exact value because the true value may not be known.

Select one:
True 

False

https://trimestral.amaesonline.com/2033/mod/quiz/review.php?attempt=165612&cmid=11785 1/13
4/24/2021 Prelim Exam: Attempt review

Question 3
Incorrect

Mark 0.00 out of 1.00

The determinant of an identity matrix is equal to 

Select one:
a. 1

b. -1 

c. 0

d. Infinite 

Your answer is incorrect.

Question 4
Correct

Mark 1.00 out of 1.00

In creating a computer algorithm one important factor that should be considered is that the user would be prompted the values that are
needed in solving.

Select one:
True 

False

Question 5
Correct

Mark 1.00 out of 1.00

If A is a 3 x 3  matrix and B is a 3x2 matrix , the statement “ A – B is not possible” is

Select one:
True 

False

https://trimestral.amaesonline.com/2033/mod/quiz/review.php?attempt=165612&cmid=11785 2/13
4/24/2021 Prelim Exam: Attempt review

Question 6
Correct

Mark 1.00 out of 1.00

 
 

Which of the following shows A + B ?

Select one:
a. No correct answer

b.

c. 

d.

Your answer is correct.

Question 7
Correct

Mark 1.00 out of 1.00

Numerical methods give more accurate results than analytic methods.

Select one:
True

False 

https://trimestral.amaesonline.com/2033/mod/quiz/review.php?attempt=165612&cmid=11785 3/13
4/24/2021 Prelim Exam: Attempt review

Question 8
Correct

Mark 1.00 out of 1.00

Creating a mathematical model that will compute the following limit as  will produce a result of

Select one:
a. infinite

b. indeterminate

c. 1 

d. 0

Your answer is correct.

Question 9
Incorrect

Mark 0.00 out of 1.00

A matrix which is denoted by a boldface lowercase letter and expressed as 1 x n matrix is

Select one:
a. no correct answer

b. row vector

c. not a matrix 

d. column vector

Your answer is incorrect.

https://trimestral.amaesonline.com/2033/mod/quiz/review.php?attempt=165612&cmid=11785 4/13
4/24/2021 Prelim Exam: Attempt review

Question 10
Correct

Mark 1.00 out of 1.00

Which of the following is true about A x B?

Select one:
a. Matrix multiplication is not possible. 

b. The resulting matrix, A x B is a 2 x 4 matrix.

c. A x B is the same as B x A.

d. All of the answers correct.

Your answer is correct.

Question 11
Incorrect

Mark 0.00 out of 1.00

The matrix defined as    is an upper triangular matrix

Select one:
True 

False

Question 12
Correct

Mark 1.00 out of 1.00

The elements of both the coefficient matrix and determinant are the same, and so is their the mathematical concept.

Select one:
True

False 

https://trimestral.amaesonline.com/2033/mod/quiz/review.php?attempt=165612&cmid=11785 5/13
4/24/2021 Prelim Exam: Attempt review

Question 13
Correct

Mark 1.00 out of 1.00

Suppose a computing machine can only display up to 4 decimal places. Assuming that the true value of π is 3.14159265359. Using an
approximate value of πa =  3.1416  Calculate the absolute error and the relative error.

Select one:
a. ɛa = 7.1126 x 10-7, %ɛ = 2.8124 x 10 -6

b. ɛa = 7.346410 x 10-7, %ɛ = 2. 3384 x 10 -6 

c. ɛa = 7.4132 x 10-7, %ɛ = 2.8243 x 10 -6

d. ɛa = 7.28754 x 10-7, %ɛ = 2.2264 x 10 -6

Your answer is correct.

Question 14
Correct

Mark 1.00 out of 1.00

The relative error is  ______________ when the exact value is given by e = 2.718281828 and the approximate value is e a = 2.701.

Select one:
a. 6.35763 x 10-3  

b. 7.55763 x 10-3

c. 5.65763 x 10-3

d. 3.65763 x 10-3

Your answer is correct.

Question 15
Incorrect

Mark 0.00 out of 1.00

The inverse of A  which is a 3 x 2  matrix is A -1 = 2 x 3 matrix.

Select one:
True 

False

https://trimestral.amaesonline.com/2033/mod/quiz/review.php?attempt=165612&cmid=11785 6/13
4/24/2021 Prelim Exam: Attempt review

Question 16
Incorrect

Mark 0.00 out of 1.00

When the exact value is π = 3.14159265359 and the relative error is given as 0.001, the approximate value is ____________.

Select one:
a. 3.141906813

b. 3.141706813

c. 3.141890681

d. 3.141606813 

Your answer is incorrect.

Question 17
Correct

Mark 1.00 out of 1.00

Interpreting the results graphically is one advantages of using software systems in numerical methods.

Select one:
True 

False

Question 18
Incorrect

Mark 0.00 out of 1.00

Which of the following is not true in finding the determinant of a matrix?

Select one:
a. No correct answer

b. Coefficients from systems of linear equations may form a matrix and could be used for finding the determinant. 

c. It doesn’t work with non-square matrix.

d. The operation between the matrices also takes an alternate sign of negative first then positive. 

Your answer is incorrect.

https://trimestral.amaesonline.com/2033/mod/quiz/review.php?attempt=165612&cmid=11785 7/13
4/24/2021 Prelim Exam: Attempt review

Question 19
Correct

Mark 1.00 out of 1.00

 
 

 
Which of the following is true about A x B?

Select one:
a. The resulting matrix A x B is a 3 x 2  matrix. 

b. All of the answers correct.

c. Matrix multiplication is not possible.

d. A x B is the same as B x A.

Your answer is correct.

Question 20
Incorrect

Mark 0.00 out of 1.00

A mathematical model that will compute the following limit as  will readily give _____________ answer.

Select one:
a. indeterminate

b. 0 

c. finite

d. infinite

Your answer is incorrect.

https://trimestral.amaesonline.com/2033/mod/quiz/review.php?attempt=165612&cmid=11785 8/13
4/24/2021 Prelim Exam: Attempt review

Question 21
Correct

Mark 1.00 out of 1.00

Perform floating point addition of 3.1 x 10 -1 and 12.25 x 10 1. If only 3 significant figures are allowed for mantissa, determine the percent
accuracy of the result.

Select one:
a. 99.01 %

b. 89.72 %

c. 99.72 % 

d. 99.88 %

Your answer is correct.

Question 22
Incorrect

Mark 0.00 out of 1.00

The absolute error of the function f(x) =  e x when the the true value of f(x) = 2.718281828 compared to the approximated value of using the
first five terms of the Maclaurin Series center at when x = 1,  c =0 is _____.

Select one:
a. 9.95 x10-3

b. 6.67 x10-3 

c. 8.83 x10-3

d. 3.35 x10-3

Your answer is incorrect.

Question 23
Correct

Mark 1.00 out of 1.00

Among the many applications of matrices are used in statistics, economics, physics, and engineering.

Select one:
True 

False

https://trimestral.amaesonline.com/2033/mod/quiz/review.php?attempt=165612&cmid=11785 9/13
4/24/2021 Prelim Exam: Attempt review

Question 24
Correct

Mark 1.00 out of 1.00

Which of the following matrices can be represented by A = 5I? (where I is the identity matrix)

Select one:
a.

b.

c. 

d.

Your answer is correct.

Question 25
Correct

Mark 1.00 out of 1.00

In floating point addition, where the exponent of the smaller number must match that of the larger number making 3.141516 x 101 and 
2.125 x 102 expressed in 3 digit precision as 0.314 x 102 and  2.13 x 102

Select one:
True 

False

https://trimestral.amaesonline.com/2033/mod/quiz/review.php?attempt=165612&cmid=11785 10/13
4/24/2021 Prelim Exam: Attempt review

Question 26
Incorrect

Mark 0.00 out of 1.00

Which of the following is true about A - B?

Select one:
a. Subtraction of matrices is not possible.

b. All of the answers correct.

c. The resulting matrix A -  B is a 3 x 2  matrix. 

d. A - B is the same as B - A.

Your answer is incorrect.

Question 27
Incorrect

Mark 0.00 out of 1.00

The determinant of the given matrix is D = |-5|.

Select one:
True 

False

https://trimestral.amaesonline.com/2033/mod/quiz/review.php?attempt=165612&cmid=11785 11/13
4/24/2021 Prelim Exam: Attempt review

Question 28
Correct

Mark 1.00 out of 1.00

The determinant of the given matrix is?

Select one:
a. 2

b. 5

c. 4

d. No correct answer 

Your answer is correct.

Question 29
Correct

Mark 1.00 out of 1.00

Which of the following shows A - B ?

Select one:
a.

b. No correct answer

c. 

d.

Your answer is correct.

https://trimestral.amaesonline.com/2033/mod/quiz/review.php?attempt=165612&cmid=11785 12/13
4/24/2021 Prelim Exam: Attempt review

Question 30
Correct

Mark 1.00 out of 1.00

The approximate value of f(x) =  e x using the first five terms of the Maclaurin Series center at when x = 1,  c =0 is

Select one:
a. 2.803

b. 2.525

c. 2.708 

d. 2.656

Your answer is correct.

◄ W3: Lesson 3 - Systems of Linear Equation - Video Lecture

Jump to...

W5: Lesson 3 - Systems of Linear Equations (Part 2) - Module ►

https://trimestral.amaesonline.com/2033/mod/quiz/review.php?attempt=165612&cmid=11785 13/13

You might also like